Categories
Exam Questions Harvard

Harvard. Introductory Economics. Exam questions, 1959-60

 

 

In 1948-49 Economics 1 replaced Economics A as the introductory course at Harvard. Really quite striking from today’s perspective are the economic history and history of economics questions from the first term and the Soviet planning questions in the second terms. You really have to be an old comparative economics researcher (or native Russian speaker) to know what “Blat” was.  [See J. Berliner, ‘Blat is higher than Stalin’, Problems of Communism, 3(1), 1954.]

__________________________

1959-60
HARVARD UNIVERSITY
ECONOMICS 1
Hour Examination.
October 30, 1959

  1. (10 minutes) Discuss briefly.
    1. Law of diminishing returns.
    2. “Favorable balance of trade.”
    3. English “enclosures.”
    4. Mercantilism
  2. (20 minutes)
    “The concept of a price system is as foreign to the economics of manorialism as the medieval doctrine of usury would be to the money markets of the modern world.”
  3. (20 minutes)
    How did the classical economists attempt to prove that there were economic gains in free trade? By what means did they translate their argument from “real” to “monetary” terms?

__________________________

1959-60
HARVARD UNIVERSITY
ECONOMICS I
Midyear Examination.
January 1960.

(Three hours)

Answer all questions

  1. (45 minutes)
    “The victory of the ‘classical economics’ over mercantilist doctrine in England, was more than anything else, an indication of the appropriateness of the new analysis to the conditions of the Industrial Revolution.”
    Discuss critically, with reference to the following:

    1. an analysis of the “classical” doctrines of growth and distribution;
    2. the areas of conflict between “classical” and mercantilist ideology;
    3. the degree of relevance of the “classical economics” to conditions in the late 18th and early 19th century England.
  2. a) Define briefly the following terms: (15 minutes)

1. The demand curve for a product
2. elasticity
3. marginal cost
4. market control or power
5. workable competition

b) Utilizing these and any other relevant concepts, discuss: (60 minutes)

1.  Price and output policy of the firm under pure competition, oligopoly, and monopoly;
2.  Government intervention in the market as exemplified in U.S. agricultural programs and proposals;
3.  The use of government anti-trust policies in improving the functioning of the market system in a less than perfectly competitive economy such as that of the U.S. today.

  1. (60 minutes) The Logic of Say’s Law (supply creates its own demand) led many economists for over 100 years to the conclusion that a general glut was impossible.
    1. How does modern macroeconomic analysis attempt to demonstrate the possibility of a general glut or depression?
    2. Can this modern analysis be used to explain the opposite situation of general excess demand? If so, how?
    3. What explanation can you give of the Great Depression by applying this analysis to specific developments in the late 1920’s and early 1930’s?
    4. What fiscal measures would you have suggested to cure the depression? Explain carefully your reason for each proposal.

 

__________________________

Economics I
Midyear Examination Make-up
April 22,1960

(Time: Three hours. Answer ALL questions.)

Question I (60 minutes)

“Markets, whether they be exchanges between primitive tribes where objects are dropped casually on the ground, or the exciting traveling affairs of the Middle Ages, are not the same as ‘the modern market system’.”

  1. What are the major economic differences between a medieval market and a modern market mechanism?
  2. How did the Mercantilist and the classical economists differ in their concepts of the role of the state with respect to the market mechanism and its operation?
  3. To what extent was the “Industrial Revolution” in England related to particular developments in the evolution of a market system?

 

Question 2 (60 minutes)

Briefly but clearly distinguish between the following pairs of concepts:

  1. marginal propensity to consume and elasticity of demand
  2. national income and national expenditure
  3. savings and investment
  4. market equilibrium and market control
  5. marginal productivity and economic rent
  6. normal profit and monopoly profit
  7. countervailing power and collective bargaining
  8. transfer payments and factor payments
  9. differentiated product and free entry
  10. perfect competition and workable competition

 

Question 3 (60 minutes)

Compare the characteristics of market structures as seen by Adam Smith with those distinguished by modern economic analysis, in relation to the following:

  1. The specific types of market structure which have developed, and the nature of the control of the individual firm with respect to its price, output, and profit.
  2. The case for intervention, by the present-day U.S. government, in
    1. particular industrial markets
    2. the agricultural market
    3. the labor market.

PLEASE RETURN THIS EXAMINATION PAPER WITH YOUR TEST BLUEBOOK

__________________________

ECONOMICS I
Section 4H [or 4II?]

Hour Examination
April 29, 1960

Answer:

Question 1;
Question 2, or Question 3; and
Question 4, or Question 5, or Question 6. A TOTAL OF THREE

  1. Comment briefly on:
    1. International Monetary Fund
    2. the marginal propensity to import and the foreign trade multiplier
    3. Gosplan
    4. Blat
    5. balance of payments deficit
    6. Soviet turnover tax
  2. Western Germany has in the past been a persistent creditor country. It has been suggested that the resulting imbalance should be remedied by an appreciation of the D-Mark.
    1. Under what circumstances would this policy decision have effective results?
    2. What set of circumstances would make such a decision undesirable?
  3. “Playing the rules of the gold standard involves a loss of freedom to monetary and fiscal authorities.” Explain.
  4. “Although prices may be used in both a planned economy and a price-directed economy, the sharpest distinction between the two can be expressed in terms of the role of prices.” Discuss.
  5. a. What is the essence of central planning? Which are the principal administrative authorities responsible for the various steps in planning in the Soviet Union?
    b. What are the principal sources of inefficiency of Soviet planning?
  6. What are some accelerating and some retarding factors affecting future Soviet Growth?

__________________________

1959-60
HARVARD UNIVERSITY
ECONOMICS 1
Final Examination.
May 25, 1960.

(Three hours)
Answer all questions

  1. (30 minutes)
    Explain what is meant by “the problem of creeping inflation.” What is the scope, and what are the limitations, of Federal Reserve policy in its attempt to meet this problem? Are there reasons for believing that mild inflation might be desirable in some respects?
  2. (30 minutes)
    The President’s Report of January 1959 gave the general fiscal objective and expectation of the Federal Government, for the fiscal year 1958-59, as a balanced budget at a level of $77 billion.
    The President’s Report of January 1960 reported that, for the fiscal year 1958-59, the Federal Government had run a deficit of approximately $12.4 billion, tax receipts being $68.3 billion and expenditures being $80.7 billion.

    1. Give a critical analysis of the major factors which tend to make budgetary planning, and fiscal policy, difficult in general, and give an account of some of the influences which threw out the predictions for the 1958-59 period in particular.
    2. What general effect, in your judgment, did the fiscal operations of government have in this period? To what extent was this effect related to deliberate policy decisions taken during the period?
  3. (30 minutes)
    The major alternatives open to the public authorities of a country experiencing a balance of payments deficit include: (a) devaluation, (b) internal deflationary policy, (c) the imposition of tariffs and import quotas.
    What are the principal economic effects associated with each of the above policies? Do you consider any of these policies appropriate for the United States in meeting its present balance of payments difficulties? Explain your answer.
  4. (45 minutes)
    In terms of achieving a rapid rate of growth of priority sectors of its economy, the U.S.S.R. has obtained what, to western economists, is a remarkable degree of success. Most of these economists, however, would distinguish this achievement from “economic efficiency.”

    1. Indicate possible sources of inefficient resource use in the Soviet system, both at the level of the firm and at the level of the higher planning authorities.
    2. Discuss the role of prices in the Soviet system in relation to the problem of an efficient allocation of resources.
  5. (45 minutes)
    “Most of mankind is caught in a vicious circle, in which poverty prevents growth and lack of growth causes poverty. Only by simultaneous, dramatic expansion of all branches of economic activity can an underdeveloped country hope to break out of this vicious circle.”
    Discuss the following, with illustrations taken from one or more underdeveloped countries:

    1. the process by which poverty and lack of growth inter-act to create a vicious circle,
    2. the validity of the contention that, to be effective, growth must be “balanced.”

 

 

Source: Harvard University Archives. Department of Economics. Course reading lists, syllabi, and exams 1913-1992 (UA V 349.295.6). Economics 1-Ec 10 exams. Box 1 of 2, Folder “Economics I, Exams 1939-1962”.

Categories
Exam Questions Harvard

Harvard. Core Economic Theory. Bullock and Carver, 1917/18 and 1918/19

 

While Frank Taussig was off serving the country as the chairman of the United States Tariff Commission, his advanced economic theory course (Economics 11) was jointly taught by his colleagues Charles Bullock and Thomas Nixon Carver. The Harvard Archives collection of final examinations only has the June final examinations that are transcribed below, i.e., the first semester examinations from January have not been included (at least for now). It is also not clear whether the course was jointly taught both semesters or whether each colleague took a semester.  The semester by semester exams for this course up to these years can be found in a series of earlier posts. Here is the most recent of the posts for Economics 11 à la Taussig up to the first term of 1916.

______________________

COURSE DESCRIPTION
(Identical for 1917/18 and 1918/19)

Primarily for Graduates
I
ECONOMIC THEORY AND METHODS

[Economics] 11. Economic Theory. Mon., Wed., Fri., at 2.30. Professors Carver and Bullock

Course 11 is intended to acquaint the student with the development of economic thought since the beginning of the nineteenth century, and at the same time to train him in the critical consideration of economic principles. The exercises are conducted mainly by the discussion of selected passages from the leading writers; and in this discussion the students are expected to take an active part. Special attention will be given to the writings of Ricardo, J.S. Mill, Cairnes, and among modern economists to F.A. Walker, Clark, Marshall, and Böhm-Bawerk.

 

Source: Official Register of Harvard University (Vol. XV, No. 23), May 10, 1918. Division of History, Government, and Economics 1918-19, p. 63.

______________________

Course Enrollment
1917-18

[Economics] 11. Professors Carver and Bullock.—Economic Theory

Total 11: 8 Graduates, 2 Graduate Business, 1 Radcliffe

 

Source: Harvard University. Reports of the President and Treasurer of Harvard College, 1917-1918, p. 54.

______________________

Final examination
(Second semester 1917-18)
ECONOMICS 11

 

  1. What elements in the economic system of Adam Smith were derived from previous systems of economic thought?
  2. What were the principal new contributions which were made by the Wealth of Nations?
  3. Trace the history of theories of business profits in the writings of Smith, Ricardo, Mill, Walker, and Marshall.
  4. Compare the theories of value presented by Smith, Ricardo, and Mill.
  5. What ideas concerning the probably future of the laboring classes were entertained by Smith, Ricardo, and Mill?
  6. Compare Smith’s views concerning rent with those of Ricardo, and then compare Mill’s statement of the theory of rent with Ricardo’s statement.
  7. What old and what new elements are found in the economic theories of Mill?
  8. From your own point of view, criticize Mill’s theory of value.

 

Source:   Harvard University Archives. Examination Papers 1918 (HU 7000.28, 60). Harvard University Examinations. Papers Set for Final Examinations in History, History of Religions, History of Science, Government, Economics,…, Fine Arts, Music in Harvard College, June, 1918, pp. 49-50.

______________________

Course Enrollment
1918-19

[Economics] 11. Professors Carver and Bullock.—Economic Theory

Total 7: 6 Civ., 1 Mil.

II, III. Total 11: 9 Graduates, 1 Senior, 1 Other.

 

Source: Harvard University. Reports of the President and Treasurer of Harvard College, 1918-1919, p. 52.

______________________

Final examination
(Second semester 1918-19)
ECONOMICS 11

Omit any one question

  1. Compare Smith’s views concerning the importance of different industries with those of the Mercantilists.
  2. What elements in the economic thought of Adam Smith were derived from earlier writers?
  3. What were Smith’s distinctive contributions to economic thought?
  4. Compare Smith’s theory of value with that of Ricardo.
  5. Compare Smith’s theory of distribution with that of Ricardo.
  6. Compare Mill’s theories of production and distribution with those of Ricardo and Smith.
  7. Give an account of the historical development of the law of diminishing returns.
  8. Compare Smith’s and Ricardo’s theories of international trade.

 

Source:   Harvard University Archives. Examination Papers 1919 (HU 7000.28, 61). Harvard University Examinations. Papers Set for Final Examinations in History, History of Science, Government, Economics,…, Fine Arts, Music in Harvard College, June, 1919, pp. 29-30.

 

 

 

 

 

Categories
Economic History Gender Harvard Radcliffe

Radcliffe. Economics courses offered by Harvard professors with descriptions, 1893-94

 

Information about economics courses offered for women by Harvard professors before Radcliffe College officially came into existence (1879-1893) were included in an earlier post. Today’s post provides course descriptions for the four course offerings in economics in Radcliffe’s first year of existence. Besides the obvious interest for the intersection of gender and history of economics, the course descriptions turn out to be more detailed in these Radcliffe Presidential Reports at this time than what I can find in the corresponding Harvard catalogues.

__________________________

Radcliffe College, 1893-94
Economics

(Primarily for Undergraduates.)

PROFESSOR [EDWARD] CUMMINGS. — Outlines of Economics. — Mill’s Principles of Political Economy. — Lectures on Economic Development, Distribution, Social Questions, and Financial Legislation. This course gave a general introduction to Economic study, and a general view of Economics sufficient for those who had not further time to give to the subject. It was designed also to give intellectual discipline by the careful discussion of principles and reasoning.

The instruction was given by question and discussion. J. S. Mill’s Principles of Political Economy formed the basis of the work. At intervals lectures were given which served to illustrate and supplement the class-room instruction. In connection with the lectures, a course of reading was prescribed. The work of students was tested from time to time by examinations and other written work. — 16 students.

 

PROFESSOR [WILLIAM JAMES] ASHLEY. — The Elements of Economic History from the Middle Ages to Modern Times. The object of this course was to give a general view of the economic development of society from the Middle Ages to the present time. It dealt, among others, with the following topics: the manorial system and serfdom; the merchant gilds and mediaeval trade; the craft gilds and mediaeval industry; the commercial supremacy of the Italian and Hanseatic merchants; the merchant adventurers and the great trading companies; the agrarian changes of the sixteenth century; domestic industry; the struggle of England with Holland and France for commercial supremacy; the beginning of modern finance; the progress of farming; the great inventions and the factory system; modern business methods; and recent financial history.

During the earlier part of the course attention was devoted chiefly to England, but that country was treated as illustrating the broader features of the economic evolution of the whole of western Europe. Arrived at the 17th century, it was shown how English conditions were modified by transference to America; and from that point an attempt was made to trace the parallel movement of English and American affairs and their mutual influence. — 8 students.

 

(For Graduates and Undergraduates.)

PROFESSOR [EDWARD] CUMMINGS. — The Principles of Sociology. — Development of Modern State, and of its Social Functions. An introductory course in sociology, intended to give a comprehensive view of the structure and development of society in relation to some of the more characteristic ethical and industrial tendencies of the present day. The course began with a theoretical consideration of the relation of the individual to society and to the state, — with a view to pointing out some theoretical misconceptions and practical errors traceable to an illegitimate use of the fundamental analogies and metaphysical formulas found in Comte, Spencer, P. Leroy Beaulieu, Schaeffle, and other writers.

The second part followed more in detail the ethical and economic growth of society. Beginning with the development of social instincts manifested in voluntary organization, it considered the genesis and theory of natural rights, the function of legislation, the sociological significance of the status of women and of the family and other institutions, — with a view to tracing the evolution of certain types of society based upon a more or less complete recognition of the social ideas already considered.

The last part dealt with certain tendencies of the modern state, discussing especially the province and limits of state activity, with some comparison of the Anglo-Saxon and the continental theory and practice in regard to private initiative and state intervention in relation to public works, industrial development, philanthrophy, education, labor organization, and the like.

Each student selected for special investigation some question closely related to the theoretical or practical aspects of the course; and a certain amount of systematic reading was expected. — 4 students.

 

(Primarily for Graduates.)

PROFESSOR [WILLIAM JAMES] ASHLEY. — Seminary in Economic History. Two students were engaged in investigations under the guidance of the Professor. Of these, one was occupied in the study of the English Poor Law and its administration, and especially of the various attempts in the sixteenth, seventeenth and eighteenth centuries to deal with the “unemployed.” The other was studying the original materials for English manorial and agrarian history in the Middle Ages; and has prepared what is believed to be a fairly exhaustive list of such materials as are already in print, classified and arranged both chronologically and topographically. This list has been published by Messrs. Ginn & Co., under the authority of Radcliffe College, as Radcliffe College Monograph. No. 6. [Frances Gardiner Davenport, A. B., A Classified List of Printed Original Materials for English Manorial And Agrarian history during the Middle Ages, prepared under the direction of W. J. Ashley, M. A., Professor of Economic History in Harvard University.]

 

Source: Radcliffe College. Reports of the President, Regent, and Treasurer 1894, pp. 49-51.

Image Source: From the cover of the 1893 Radcliffe Yearbook.

 

 

Categories
Uncategorized

Harvard. Alumnus Economics Ph.D. Canadian Prime Minister, W. L. Mackenzie King. 1909

 

 

In the continuing series, Get to Know an Economics Ph.D., we meet W. L. Mackenzie King who made his way to Harvard via the Universities of Toronto and Chicago. In a quick internet background check I stumbled upon the fact that this most distinguished Harvard alumnus was also someone who consulted crystal balls and regularly attended seances to communicate with the dead. These odd  facts only became known upon the publication of his diaries that can be consulted online at the Libaries and Archives Canada website. In this post I provide a 1921 report from the Harvard Alumni Bulletin and the excerpt from his diary describing his first day at Harvard.

Dictionary of Canadian Biography Vol. XVII (1941-1950).
Library and Archives Canada. Diaries of William Lyon Mackenzie King.

________________________

A Harvard Prime Minister.
W. L. Mackenzie King, A.M. 1898, Ph.D. 1909

To W. L. Mackenzie King, Prime Minister. A.M. ’98, Ph.D. ’09, who led the Liberals to victory in the recent Canadian elections and now becomes Prime Minister of the Dominion, we are glad to extend our felicitations. Harvard men have had seats in the Cabinet at Ottawa on several previous occasions, but this is the first time that the University has had one of her graduates at the head of the ministerial table there.

After taking his undergraduate course at the University of Toronto, Mr. King came to Harvard in September, 1897. He had already acquired an interest in labor problems and his chief purpose in coming to Cambridge was to study economics under the guidance of Professor Taussig. His strength of intellect and his vigorous personality soon made an impression, both in the classroom and out of it; Mr. King was generally regarded as one of the ablest among the graduate students of his day. After receiving his master’s degree in 1898 and spending a further year in work leading to the Ph.D. degree in economics, he was awarded the Henry Lee Memorial Fellowship for study abroad, and spent the winter of 1899-1900 in Europe, chiefly at the London School of Economics.

Returning from London in the summer of 1900, Mr. King was offered an instructorship in economics at Harvard and was on the point of accepting it when a new development caused him to shift his plans. The Canadian government, just at this time, had decided to establish a Labor Bureau, and, casting about for a capable young man to take charge of it, offered the post to Mr. King, who accepted it with some misgivings because he had set his ambitions upon an academic career. At any rate, the Department of Economics had to find another instructor while King went to Ottawa where he became Deputy Minister of Labor and filled that post with such conspicuous success that in 1909 he was made a member of the Laurier Cabinet. With the defeat of the Liberals in 1911 he went out of office, but not out of public life, and on the death of Sir Wilfred Laurier he became the leader of the Liberal party. Now, at the age of forty-seven, he has reached the top rung of his political ladder.

During the twenty years since he left the University, Mr. King has never lost touch with Harvard. Among Harvard graduates in Canada he has been one of the most loyal and enthusiastic. From time to time he has looked in upon us during brief visits to Boston and we count him no stranger here. He has our best wishes for a successful administration.

Source: Harvard Alumni Bulletin. Vol. XXIV, No. 13 (December 22, 1921), pp. 285-286.

________________________

 First Day at Harvard
from W. L. Mackenzie King’s Diary

Tuesday, September 28, 1897.
(Handwritten—p. 271)

…We arrived at Boston about 10.30 got off at Huntingdon St. & was greatly taken with the first glimpse at the city, everything so clear, like an English city, the names etc. not so much smoke or rush, took a street car for Harvard square & went at once to the College yard.

After looking about for a while I met Prof. Ashley & gave him the letter from Mr. Smith. He was very kind in showing me the library & taking me to the Episc. Theol. Seminary, where he sd. he hoped I wd. get a room as the life there was more like the Oxford life than anything in Cambridge. He tried to find the Bursar, pointed out to me the Washington Elm & the Craigie House, which are both close by, told me about other points of interest. I am much taken with the beauty of Cambridge, it is an ideal college town—the trees are beautiful, people live as well as exist here. I met Mr. Jno. Cummings during the afternoon. He is quite a young fellow & not just the man I expected. Spent a good part of the afternoon waiting upon the Bursar of the Epis. Theol. Seminary. May be able to get a room there. Went on a search for other rooms. About tea time got a place at 14 Millar St., Mrs. Morrison, where I could room & board till I get settled. The woman told me all her sorrows at the table. After dinner, after helping some girls with a bicycle, I called on Prof. Taussig—short call. I went for a stroll after dinner met a young girl was anxious to talk with some one lonely a little & restless. We walked about awhile together. Had I read my diary before going out I wd. not have gone so far—returned about 10. Went to bed tired & sorry at the mistake I had made.

Met Prof. Peabody who was very kind in offering to be of any service to me. Prices are very high here. Saw a room on 14 Sumer St. I like very much too high, 150 wd. come down 10.

Source:  Libraries and Archives Canada. Diaries of Prime Minister William Lyon Mackenzie King, MG26-J13. Item: 1650.

Image Source: William Lyon Mackenzie King in 1899. Wikipedia.

Categories
Exam Questions Harvard

Harvard. Core Advanced Economic Theory. Taussig (and Day), 1915-1917

 

 

Examination questions spanning just over a half-century can be found in Frank Taussig’s personal scrapbook of cut-and-pasted semester examinations for his entire Harvard career. Until Schumpeter took over the core economic theory course from Taussig in 1935, Taussig’s course covering economic theory and its history was a part of almost every properly educated Harvard economist’s basic training. Taussig’s exam questions were posted for the academic years 1886/87 through 1889/90 along with enrollment data for the course;  material from 1890/91 through 1893/94; 1897-1900 ; 1904-1909 ; 1911-14 have been posted as well.  

The course was taught by Taussig up through the Winter term of 1916/17. Early in 1917 Taussig was appointed chairman of the newly created United States Tariff Commission. He also was appointed a member of the Advisory Committee on the Peace (sub-committee on tariffs and commercial treaties) and he went to Europe for the economic sessions of the peace negotiations. His resignation from the Tariff Commission was  effective August 1, 1919 after which he returned to Harvard.

U.S. Tariff Commission Reports under Taussig 1917-1919:

First Annual Report (Fiscal Year ended June 30, 1917)
Second Annual Report (Fiscal Year ended June 30, 1918)
Third Annual Report (1919). 

________________________________

1914-15
HARVARD UNIVERSITY

ECONOMICS 11
Mid-year Examination
[F. W. Taussig]

Arrange your answers in the order of the questions.

  1. “Given machinery, raw materials, and a year’s subsistence for 1000 laborers, does it make no difference with the annual product whether those laborers are Englishmen or East Indians?
    . . . The differences in the industrial quality of distinct communities of laborers are so great as to prohibit us from making use of capital to determine the amount that can be expended in any year or series of years in the purchase of labor.”
    Under what further suppositions, if under any, does this hypothetical case tell in favor of those holding that wages are paid from a wages fund? Under what suppositions, if under any, in favor of those holding views like Walker’s?
  2. (a)“The labourer is only paid a really high price for his labour when his wages will purchase the produce of a great deal of labour.”
    (b) “If I have to hire a labourer for a week, and instead of ten shillings I pay him eight, no variation having taken place in the value of money, the labourer can probably obtain more food and necessaries with his eight shillings than he before obtained for ten.”
    Explain concisely what Ricardo meant.
  3. What, according to Ricardo, would be the effects of a general rise of wages on profits? on the prices of commodities? on rents? the well-being of laborers?
  4. “The component elements of Cost of Production have been set forth in the first part of this enquiry. The principal of them, and so much the principal as to be nearly the sole, we found to be Labour. What the production of a thing costs to its producer, or its series of producers, is the labour expended in producing it. If we consider as the producer the capitalist who makes the advances, the word Labour may be replaced by the word Wages: what the produce costs to him, is the wages which he has had to pay.”   J.S. Mill.
    What would Ricardo say to the proposed substitution [of “Wages” for “Labour”]? Cairnes? Marshall?
  5. “Suppose that society is divided into a number of horizontal grades, each of which is recruited from the children of its own members, and each of which has its own standard of comfort, and increases in number rapidly when the earnings to be got in it rise above, and shrinks rapidly when they fall below that standard. Suppose, then, that parents can bring up their children to any trade in their own grade, but cannot easily raise them above it and will not consent to sink them below it. . . .
    On these suppositions the normal wage in any trade is that which is sufficient to enable a laborer, who has normal regularity of employment, to support himself and a family of normal size according to the standard of comfort that is normal in the grade to which his trade belongs. In other words the normal wage represents the expenses of production of the labor according to the ruling standard of comfort.” Marshall.
    On these suppositions, would value depend in the last analysis on cost or utility?
  6. (a)“Were it not for the tendency [to diminishing returns] every farmer could save nearly the whole of his rent by giving up all but a small piece of his land, and bestowing all his labor and capital on that. If all the labor and capital which he would in that case apply to it gave as good a return in proportion as that he now applies to it, he would get from that plot as large a produce as he now gets from his whole farm; and he would make a net gain of all his rent save that of the little plot that he retained.”
    (b) “The return to additional labour and capital [applied to land] diminishes sooner or later; the return is here measured by the quantity of the produce, not by its value.”
    (c) “Ricardo, and the economists of his time generally were too hasty in deducing this inference [tendency to increased pressure] from the law of diminishing return; and they did not allow enough for the increase of strength that comes from organization. But in fact every farmer is aided by the presence of neighbours, whether agriculturists or townspeople. . . . If the neighbouring market town expands into a large industrial centre, all his produce is worth more; some things which he used to throw away fetch a good price. He finds new openings in dairy farming and market gardening, and with a larger range of produce he makes use of rotations that keep his land always active without denuding it of any one of the elements that are necessary for its fertility.”
    Have you any criticisms or qualifications to suggest on these passages from Marshall?
  7. “When the artisan or professional man has once obtained the skill required for his work, a part of his earnings are for the future really a quasi-rent. The remainder of his income is true earnings of effort. But this remainder is generally a large part of the whole. And herein lies the contrast. When a similar analysis is made of the profits of the undertaker of business, the proportions are found to be different: in this case nearly all is quasi-rent.”
    Explain what you believe to be Marshall’s meaning, and why he considers undertaker’s profits not to be “true earnings of effort.”

________________________________

1914-15
HARVARD UNIVERSITY

ECONOMICS 11
Final Examination
[F. W. Taussig]

Arrange your answers in the order of the questions.

  1. Explain briefly what Walker meant by the “no-profits” business man; what Marshall means by the “representative firm”; what your instructor means by the “marginal product of labor.” How are the three related?
  2. Explain briefly whether anything in the nature of a producer’s surplus or a consumer’s surplus appears as regards (a) instruments made by man and the return secured by their owners; (b) unskilled labor and the wages paid for it; (c) business management and business profits.
  3. “ Wages are paid by the ordinary employer as the equivalent of the discounted future benefits which the laborer’s work will bring him — the employer — and the rate he is willing to pay is equal to the marginal desirability of the laborer’s services measured in present money. We wish to emphasize the fact that the employer’s valuation is (1) marginal, and (2) discounted. The employer pays for all his workmen’s services on the basis of the services least desirable to him, just as the purchaser of coal buys it all on the basis of the ton least desirable to him; he watches the ‘marginal’ benefits he gets exactly as does the purchaser of coal. At a given rate of wages he ‘buys labor’ up to the point where the last or marginal man’s work is barely worth paying for. . . . If, say, he decides on one hundred men as the number he will employ, this is because the hundredth or marginal man he employs is believed to be barely worth his wages, while the man just beyond this margin, the one hundred and first man, is not taken on because the additional work he would do is believed to be not quite worth his wages.”
    Does this seem to you in essentials like the doctrine of Clark? of your instructor?
    [Hand-written note: The author is I. Fisher.]
  4. An urban site is leased at a ground rental of $2,000 a year; a building is erected on it costing $50,000; the current rate of interest is 4%.
    Suppose the net rental of the property (after deduction of expenses and taxes) to be $8,000. What is the nature of this return, according to J. S. Mill? Marshall? Clark?
    Suppose the net rental to be $3,000; answer the same questions.
  5. “That capital is productive has often been questioned, but no one would deny that tools and other materials of production are useful; yet these two propositions mean exactly the same when correctly understood. Capital consists primarily of tools and other materials of production, and such things are useful only in so far as they add something to the product of the community. Find out how much can be produced without any particular tool or machine, and then how much can be produced with it, and in the difference you have the measure of its productiveness.”
    What would Böhm-Bawerk say to this? What is your own view?
    [Hand-written note: The author is Carver.]
  6. “ Wages bear the same relation to man’s services that rent does to the material uses of wealth. . . . While rent is the value of the uses of things, wages is the value of the services of men. . . . The resemblance is very close between rent and wages.”
    “The principles governing the rate of wages are, in a general way, similar to those governing the rate of rent. The rate of a man’s wages per unit of time is the product of the price per piece of the work he turns out multiplied by the rate of output. His productivity depends on technical conditions, including his size, strength, skill, and cleverness.”
    Explain what is meant by “rent” in these passages and by what writers it is used in this sense; and give your opinion on the resemblance between such “rent” and wages.
    [Hand-written note: The authors are Fetter and Fisher, respectively.]
  7. Böhm-Bawerk remarks that the theory which he has put forward bears “a certain resemblance” to the wages fund theory of the older English school, but differs from it in various ways, one of which is “the most important” What are the points of resemblance? and what is this most important difference?
  8. “While the slowness of Nature is a sufficient cause for interest, her productivity is an additional cause. . . . Nature is reproductive and tends to multiply. Growing crops and animals make it possible to endow the future more richly than the present. By waiting, man can obtain from the forest or farm more than he can by premature cutting or the exhaustion of the soil. In other words, not only the slowness of Nature, but also her productivity or growth, has a strong tendency to keep up the rate of interest. Nature offers man, as one of her optional income-streams, the possibility of great future abundance at trifling present sacrifice. This option acts as a bribe to man to sacrifice present income for future, and this tends to make present income scarce and future income abundant, and hence also to create in his mind a preference for a unit of present over a unit of future income.”
    What would Böhm-Bawerk say to this? What is your own view?
    Whom do you believe to be the writer of the passage?
    [Hand-written note: The author is I. Fisher]

________________________________

1915-16
HARVARD UNIVERSITY

ECONOMICS 11
Mid-year Examination
[F. W. Taussig]

Arrange your answers strictly in the order of the questions.

  1. On what grounds is it contended that there is a circle in Walker’s reasoning on the relation between wages and business profits? What is your opinion on this rejoinder: that Walker, in speaking of the causes determining wages, has in mind the general rate of wages, whereas in speaking of profits he has in mind the wages of a particular grade of labor?
  2. According to Ricardo, neither profits of capital nor rent of land are contained in the price of exchangeable commodities, but labor only.” — Thünen.
    Is there justification for this interpretation of Ricardo?
  3. “Instead of saying that profits depend on wages, let us say (what Ricardo really meant) that they depend on the cost of labour. . . . The cost of labour is, in the language of mathematics, a function of three variables: the efficiency of labor; the wages of labour (meaning thereby the real reward of the labourer); and the greater or less cost at which the articles composing that real reward can be produced or procured.”   — J. S. Mill.
    Is this what Ricardo really meant? Why the different form of statement by Mill? What comment have you to make on Mill’s statement?
  4. State resemblances and differences in the methods of analysis, and in the conclusions reached, between (a) the temporary equilibrium of supply and demand (e.g. in a grain market), as explained by Marshall; (b) “two-sided competition,” as explained by Böhm-Bawerk; (c) equilibrium under barter, as explained by Marshall.
  5. Explain concisely what is meant in the Austrian terminology by “value,” “subjective value,” “subjective exchange value,” “objective exchange value.”
    Does the introduction of “subjective exchange value” into the analysis of two-sided competition lead to reasoning in a circle?
  6. “Suppose a poor man receives every day two pieces of bread, while one is enough to allay the pangs of positive hunger, what value will one of the two pieces of bread have for him? The answer is easy enough. If he gives away the piece of bread, he will lose, and if he keeps it he will secure, provision for that degree of want which makes itself felt whenever positive hunger has been allayed. We may call this the second degree of utility. One of two entirely similar goods is, therefore, equal in value to the second degree in the scale of utility of that particular class of goods. . . . Not only has one of two goods the value of the second degree of utility, but either of them has it, whichever one may choose. And three pieces have together three times the value of the third degree of utility, and four pieces have four times the value of the fourth degree. In a word, the value of a supply of similar goods is equal to the sum of the items multiplied by the marginal utility.” — Wieser.
    Do you think this analysis tenable? and do you think it inconsistent with the doctrine of total utility and consumer’s surplus?
  7. “If the modern theory of value, as it is commonly stated, were literally true, most articles of high quality would sell for three times as much as they actually bring.” What leads Clark to this conclusion? and do you accept it?

________________________________

1915-16
HARVARD UNIVERSITY

ECONOMICS 11
Final Examination
[F. W. Taussig]

Arrange your answers strictly in the order of the questions. Allow time for careful revision of your answers.

  1. “The productivity of capital is, like that of land and labor, subject to the principle of marginal productivity, which is, as we have seen, a part of the general law of diminishing returns. Increase the number of instruments of a given kind in any industrial establishment, leaving everything else in the establishment the same as before , and you will probably increase the total product of the establishment somewhat, but you will not increase the product as much as you have the instruments in question. Introduce a few more looms into a cotton factory without increasing the labor or the other forms of machinery, and you will add a certain small amount to the total output. . . . That which is true of looms in this particular is also true of ploughs on a farm, of locomotives on a railway, of floor space in a store, and of every other form of capital used in industry.” Is this in accord with Clark’s view? Böhm-Bawerk’s? Marshall’s? Your own?
  2. What is the significance of the principle of quasi-rent for
    1. the “single tax” proposal;
    2. Clark’s doctrine concerning the specific product of capital;
    3. the theory of business profits.
  3. Explain what writers use the following terms and in what senses: Composite quasi-rent; usance; implicit interest; joint demand.
  4. On Cairnes’ reasoning, are high wages of a particular group of laborers the cause of the result of high value (price) of the commodities made by them? On the reasoning of the Austrian school, what is the relation between cost and value? Consider differences or resemblances between the two trains of reasoning.
  5. “This ‘exploitation of interest’ consists virtually of two propositions: first, that the value of any product usually exceeds its cost of production; and, secondly, that the value of any product ought to be exactly equal to its cost of production. The first of these propositions is true, but the second is false. Economists have usually pursued a wrong method in answering the socialists, for they have attacked the first proposition instead of the second. The socialist is quite right in his contention that the value of the product exceeds the cost. In fact, this proposition is fundamental in the whole theory of capital and interest. Ricardo here, as in many other places in economics, has been partly right and partly wrong. He was one of the first to fall into the fallacy that the value of the product was normally equal to its cost, but he also noted certain apparent ‘exceptions,’ as for instance, that wine increased in value with years.” Is this a just statement of Ricardo’s view? Of the views of economists generally? In what sense is it true, in in any, that value usually exceeds cost?
  6. Explain carefully what Böhm-Bawerk means by
    1. social capital;
    2. the general subsistence fund;
    3. the average production period;
    4. usurious interest.
  7. In what way does he analyze the relation between (b) and (c)?
  8. Suppose ability of the highest kind in the organization and management of industry became as common as ability to do unskilled manual labor is now; what consequences would you expect as regards the national dividend? the remuneration of the business manager and of the unskilled laborer? Would you consider the readjusted scale of remuneration more or less equitable that that now obtaining?
  9. What grounds are there for maintaining or denying that “profits” are (a) essentially a differential gain, (b) ordinarily capitalized as “common stock,” (c) secured through “pecuniary,” not “industrial” activity? What method of investigation would you suggest as the best for answering these questions?

________________________________

Course Enrollment
1916-17

[Economics] 11. Asst. Professor Day.—Economic Theory

Total 28: 21 Graduates, 2 Seniors, 1 Junior, 1 Radcliffe, 3 Others

Source: Harvard University. Reports of the President and Treasurer of Harvard College, 1916-1917, p. 57.

________________________________

1916-17
HARVARD UNIVERSITY

ECONOMICS 11
Midyear Examination
[F. W. Taussig]

Arrange your answers strictly in the order of the questions.

  1. “Is it not true, in any normal condition of things, that consumption is supported by contemporaneous production?
    . . . Just as the subsistence of the laborers who built the Pyramids was drawn not from a previously hoarded stock, but from the constantly recurring crops of the Nile Valley; just as a modern government when it undertakes a great work of years does not appropriate to it wealth already produced, but wealth yet to be produced, which is taken from producers in taxes as the work progresses; so is it that the subsistence of the laborers engaged in production which does not directly yield subsistence, comes from the production of subsistence in which others are simultaneously engaged.”
    Consider, as regards contemporaneous production in general and also as regards the example of the Pyramids.
  2. “Our [British] commodities would not sell abroad for more or less in consequence of a free trade and a cheap price of corn; but the cost of production to our manufacturers would be very different if the price of corn was eighty or was sixty shillings per quarter; and consequently profits would be augmented by all the cost saved in the production of exported commodities.” — Ricardo.
    Explain what Ricardo meant here by “cost of production”; why he thought cost would be different in consequence of free trade in corn; and whether he believed cost (in this sense) to be the regulator of value.
  3. In what sense is the term “demand” used by Mill when speaking of (a) the equation of demand and supply, (b) demand and supply in relation to labor, (c) the demand for money?
  4. “The one universal rule to which the demand curve conforms is that it is inclined negatively throughout the whole of its length.” Can you mention exceptions as regards the demand curve for short periods? for long periods? In what sense is the term “demand” here used?
  5. It has been said that Marshall’s discussion of demand and utility is “an elementary analysis of an almost purely formal kind.” Does this seem to you a just comment?
  6. Explain “subjective value” and “subjective exchange value.” Under what conditions is subjective value to sellers of substantial influence in the determination of “objective exchange value”? Under what conditions, if under any, is subjective exchange value effective in such determination?
  7. “He [Longe] puts the case of a capitalist who, by taking advantage of the necessities of his workmen, effects a reduction in their wages, and succeeds in withdrawing so much, call it £1000, from the wages-fund; and asks how is the sum, thus withdrawn, to be restored to the fund? On Mr. Longe’s principles the answer is simple — ‘by being spent on commodities;’ for it may be assumed that the sum so withdrawn will, in any case, not be hoarded. . . . And I am disposed to flatter myself that the reader who has gone with me in the foregoing discussion will not have much difficulty in replying to it [the question] upon mine.”
    What is the answer on Cairnes’s principles? and is this the answer to be expected on the basis of a wages-fund doctrine?
  8. Explain in what way the relation between cost and value is analyzed by Cairnes and by the Austrian School. Would Cairnes’s analysis differ in essentials from the Austrian, if he were to assume complete mobility of labor? What significance do the Austrians attach to mobility of labor?

________________________________

1916-17
HARVARD UNIVERSITY

ECONOMICS 11
Final Examination
[E. E. Day]

  1. “The ultimate determinant of value…is marginal utility, not cost in the sense of labor of effort.” What would Marshall say of this? Böhm-Bawerk? Taussig?
  2. “The forces which make for Increasing Return are not of the same order as those that make for Diminishing Return…The two ‘laws’ are in no sense coordinate….The two ‘laws’ hold united, not divided, sway over industry.” Comment critically.
  3. Suppose the Federal government imposes a tax of 10 cents a bushel on all wheat grown in the United States. Upon whom will the burden of the tax fall? What conditions determine the final incidence of the tax? Illustrate, where possible, by diagram.
  4. “Rent forms no part of the expenses of production….Rent is not one of the factors bearing on price, but is the result of price.” Carefully analyze this contention.
  5. “The differences in the productive power of men due to their heredity or social position give to certain individuals the same kind of an advantage over others that the owner of a corner lot in the center of a city has over one in the suburbs. If the income from a corner lot is a surplus and can therefore be described as unearned, the income of a man of better heredity, education or opportunity must also be regarded as a surplus income and therefore unearned.” Discuss this statement with reference to your general theory of distribution.
  6. Contrast briefly the definitions of “capital” advanced by (a) Böhm-Bawerk; (b) Clark; (c) Taussig; (d) Fetter; (e) Veblen.
  7. Discuss the place of abstinence (or the sacrifice of saving) in the interest theories of (a) Böhm-Bawerk, (b) Clark; (c) Fetter; (d) Taussig.
  8. “In previous chapters, interest has been accounted for, in part at least, by the fact that there is productivity of capital; it results from the application of labor in more productive ways. If this were the whole of the theory of interest, we should reason in a circle in saying that wages are determined by a process of discount.” Do you agree as to the circle? Why or why not?

Source: Harvard University Archives. Examination Papers (HUC 7000.28, vol. 59). Harvard University Examinations. Papers Set for Final Examinations in History, History of Religions, History of Science, Government, Economics,…, Fine Arts, Music in Harvard College, June 1917. p. 61.

Image Source: Frank Taussig’s 1919 passport application.

________________________________

Categories
Harvard Social Work

Harvard. Interdisciplinary Department of Social Ethics, 1920

 

The death of the benefactor of Harvard’s Department of Social Ethics, Alfred Tredway White (1846-1921), provided the Harvard Alumni Bulletin an opportunity to review the history of the origins and progress of the interdisciplinary Department of Social Ethics established in 1905 which could trace some of its roots to the sociology course offerings of the Department of Economics. 

________________________

Social Ethics.

The article by Professor Cabot which we print in the present issue serves a double purpose. On the one hand it pays a fitting tribute to the memory of one of Harvard’s most generous and self-forgetful benefactors. Mr. Alfred T. White did not give from love of himself, nor even from love of something that was his, such as an alma mater. He gave to a cause in which he believed, and he was concerned only that that cause might be effectively promoted.

But Professor Cabot’s article also throws light on the history and plans of one of the most interesting departments of the University. There is a sense in which this light is needed—for the Department suffers from its ambiguity. It has grown up in close relations with Philosophy, and is at present a member of the same division, and a fellow-tenant of Emerson Hall. Furthermore, Social Ethics sounds like “ethics”, and it is well known that ethics is a branch of philosophy. On the other hand, Social Ethics sounds almost equally like sociology; and that, according to our Harvard plan of organization, is a branch or dependency of Economics. Furthermore, when we come to examine the details of the Social Ethics courses we find that they deal with poverty, immigration, labor, and the like; and these topics appear also in the courses on Economics. There is even a third affinity that confuses the identity of Social Ethics. It is edifying Social Ethics. and improving, and in that respect like Divinity. When Professor Peabody headed the Department of Social Ethics he was at the same time “Plummer Professor of Christian Morals” and preached (as happily he still does) in Appleton Chapel.

What, then, would be left of Social Ethics if its definitions of moral standards were assigned to Philosophy, its descriptions of social facts to Economics, and its devotional spirit to the Divinity School? Nothing—that is, nothing except just that peculiar thing which you get when the three are combined. But the more one thinks of it the more clear one becomes that they are well worth combining.

Consider, for example, the case of poverty. The mere philosopher will prove that it is evil; the mere economist will describe its quantity, its varieties, and its causes; the mere priest will visit the poor and pity them. But suppose you combine the three things in one and the same man. He will have a rational and defensible judgment that poverty is bad; he will be well-informed about it, especially in its broader aspects and underlying conditions; and he will seek to provide a remedy. Now it was Professor Peabody‘s idea and Mr. White’s idea that society will be best served by this thrice-armed man, and that it might well be one of the functions of a great university to arm him and send him forth.

That every college man should acquire something of this reasoned and enlightened zeal to help effectively in the ceaseless struggle of man against nature and against his own infirmities, it would indeed be cynical to doubt. That there should be a special Department of the University in which this three-fold interest is focussed and nurtured is fitting and desirable. But apart from this contribution to undergraduate instruction, the Department of Social Ethics promises to render an important service to the community at large in its development of instruction for professional social workers. Several such courses are announced in the new pamphlet for 1921-22 as offered by the Department itself. But more significant of future development and possibilities is the reference to courses offered in other Departments or schools of the University, which by being systematically grouped would serve as admirable programs of professional social training. Thus, for example, courses in Social Ethics and Education (courses on play, mental hygiene, etc.) make up a varied and adequate program for workers in community centres, settlement houses, or recreation departments. It is evident in this case as doubtless in many others that the rich resources of the University may be made to serve new ends merely through being intelligently correlated with one another and with the public needs of the time.

________________________

A. T. White and the Department of Social Ethics

By Richard C. Cabot, ’89, Professor of Clinical Medicine and Professor of Social Ethics

Alfred T. White of Brooklyn, N. Y., has been the benefactor of the Department of Social Ethics at Harvard. His recent death makes it fitting to sum up here and now what he has done for the University.

Other benefactors have given to Harvard larger sums. But seldom has a single department been so generously and so steadily supported by a single individual. The total amount of his gifts has now reached nearly $283,000. In 1903 he gave $50,000 to provide quarters for Social Ethics in the new Philosophy Building then projected. In 1905 he added $100,000 as an endowment of the Department. In 1917 and again in 1918 he gave $50,000 for the same purpose. His will contained a bequest for $50,000, to which should be added smaller donations for temporary needs.

In these gifts there are several unusual qualities. First,—the giver was not a Harvard graduate. He was moved to help social ethics because he believed in it and because he believed in Professor F. G. Peabody, his life-long friend. Moreover, Mr. White believed in social ethics when almost no one else did. Professor Peabody has recently pointed this out: “When Mr. White began to invest in the teaching of social ethics at Harvard University, the subject was hardly recognized as appropriate to a place of learning and was viewed by many critics with apprehension and by some with hostility. Mr. White, however, realized that the problems of social welfare and change must be, as he once said, the central matter of interest to educated .young men for the next fifty years. He proceeded to create what was, I believe, the first systematic and academic department for such instruction that this or any other University has maintained.”

Moreover, he was a remarkably persistent giver. “It was a dramatic opportunity,” says Professor Peabody, “to endow a department of social ethics, but it was a much severer test of conviction to be the anonymous source of a continuous stream of benefactions, prizes, publications, and equipment for nearly twenty years and to secure their continuance after his death.”

I do not wish to prescribe a precise application for every part of the income which will arise from this endowment, but I shall be glad to have it applied toward the provision and maintenance of material, such as books, photographs, drawings, models, etc., toward a special library and a social museum; toward the payment of further instructors, assistants, and curators; to the encouragement through prizes, fellowships, and other rewards, of special researches or publications; or for lectures or new forms of instruction. My interest in developing these studies at Harvard University is prompted largely by my observation of the courses originated and directed by Professor Peabody, and it is my desire that, while he continues to administer this instruction, the income from this endowment shall be expended, with the concurrence of the Corporation, under his direction and in fulfillment of the purposes which he has in mind. I would like to have the endowment known as “The Francis Greenwood Peabody Endowment” for the encouragement of the studies of the Ethics of the Social Questions.

Doubtless the adventurous and pioneering quality of Mr. White’s gifts was enhanced by the fact that he was helping another pioneer. For Professor Peabody’s courses anticipated by many years the earliest teaching of social work in this country. The Boston School for Social Workers, one of the earliest in the country, was not founded until 1904—or twenty-two years after the time when Professor Peabody began to give similar instruction at Harvard.

It was in the autumn of 1883 that there first appeared as Philosophy II (later Philosophy 5) a course by Professor Francis G. Peabody described as: “Ethical Theories and Moral Reforms. Studies of the practical problems of temperance, charity, divorce, the Indians, labor, prison discipline, etc.” —a half-course. This course, to which there was added in 1895 a Seminary in Sociology (200), was given by Professor Peabody both in the Divinity School and in the Philosophical Department up to 1905, a period of twenty-two years. In 1904, Dr. Jeffrey R. Brackett, of the newly established Boston School for Social Workers, began to give also (as Philosophy 19) a course on “The Practical Problems of Charity, Public Aid and Correction”.

These courses, which at their inception had no parallels in any other American college, attracted the interest of Mr. White, long an intimate and valued friend of Professor Peabody. The result is best stated in his own words:

For fifty years my approach to any understanding of the involved social and industrial problems of the day has been from the point of view and practical experience of a layman. It was a recognition of a dire need which led me more than forty years ago to endeavor to study housing problems, but I was forced to cross the Atlantic to obtain any guidance. Incidentally, I became interested in industrial problems, in problems of intemperance, etc. . . . . When I found some thirty years since that Professor Peabody was endeavoring to instruct classes at Harvard along the very lines on which I had been endeavoring to work or find guidance, it seemed to me that an opportunity was presented of which it was my duty to make the most, and my contribution to the erection of Emerson Hall and the endowment of the Department of Social Ethics resulted.

This result was attained in 1905, when the Department of Social Ethics first appears in the University Catalogue, following that of Philosophy, and began to occupy its present quarters on the second floor of Emerson Hall, where space was provided (according to the plan of Professor Peabody and Mr. White) for a museum of social ethics and for a social ethics library, as well as for recitation rooms and small departmental study-rooms. Mr. White hoped that in this new building the Department might extend its usefulness and its influence:

I wish that all the teaching in the Department of Social Ethics might be of the highest possible quality, but I wish also that the Department might be made to reach the largest possible number of undergraduates. During fifty years I have seen the difficulty of making sane progress which is due largely on the one side to satisfied ignorance and on the other to untrained theorists. Instruction which Harvard has given and is giving in its Department of Social Ethics in the way of promoting careful and sane consideration of social and industrial problems seems to me really invaluable. Not infrequently I have happened to hear testimonies to its great usefulness.

It now seems clear to me that instruction in these subjects of study will have an unprecedented opportunity of usefulness in connection with the consideration of the grave problems of reconstruction which are opening before this country.

At the close of the Civil War I rejoiced to be coming of age at a time when similar though lesser problems confronted us, and now I am almost envious of those who are coming to manhood at this time and of those who have the opportunity to instruct them.

In accordance with these hopes, the Department added to its staff in 1908 Doctors Ford, Foerster, and McConnell, the first two of whom, after Professor Peabody’s retirement in 1913, have carried on the courses up to the present academic year.

The group of subjects which Professor Peabody could treat in the Department’s early years under the compass of a single course (at first a half-course) have since then been developed and separated into two separate full courses and nine half-courses. Thus Dr. Rogers (1905) and later Dr. McConnell gave separate half-courses in “Criminology and Penology”. The “European Phases of Social Effort” needed special treatment in a half-course by Dr. Foerster, begun in 1909. “Rural Social Development” (Dr. Ford) was added next year, “Housing Problems” (Dr. Ford) in 1912 and a new course, “Immigration and Race Problems”, by Dr. Foerster appears in the same year. In 1913 the “Alcohol Problem” becomes under Assistant Professor Ford a topic deserving separate treatment, and Mr. Carstens comes in from his Boston work in the Prevention of Cruelty to Children to give a course in “Child Helping Agencies”.

Hitherto all the ethical problems involved in the “Labor Question” had been treated as part of the general introductory course with which Professor Peabody began. In 1915 another offshoot appears as Social Ethics 6,—“Unemployment and other interruptions of income with special reference to social insurance” (Professor Foerster), also a seminary in “labor legislation, standards of living and earning”. In 1916 “Poor Relief” becomes a separate half-course under Assistant Professor Ford, and Assistant Professor Foerster adds a half-course in “Recent Theories of Social Reform”.

In 1920 the courses fitted to train professional social workers were separated from the rest as definitely professional courses, carried on by Professor Ford. An introductory course (A) and another advanced course (16) have also been added.

Mr. White assigned a very central position to the study of social ethics. He believed, as I do, that social ethics differs from most other subjects in being one that only an automaton or a maniac can wholly neglect. To direct one’s affairs at all, one must make some estimate of a better and a worse, which estimate is ethical and almost invariably social. One can neglect music and mathematics, chemistry and Latin, history and economics, if one is so foolish. But even neglect and foolishness have an ethical tinge in all but the most hare-brained people.

In one sense, then, social ethics is a subject that everyone deals with, well or ill. In this sense, like language, it is everybody’s specialty. But the question remains: Can social ethics be taught? I do not know whether Mr. White ever asked himself this question. I admit that it seems to me difficult to answer it with a confident affirmative. Each of us must, to a large extent, teach himself and find his own way in ethics. But this is almost as true of every other important subject. Only the mechanical and mnemonic elements of music, history, or mathematics can be “taught”. The spirit of these studies and of all studies has to be found by each for himself. This belief is, I suppose, at the root of President Lowell’s advocacy of the tutorial system. How to find out for oneself the interest of any study is perhaps possible under tutorial guidance for many who never could discover it in the class room. At any rate our chance of usefulness to the student will be as good as anyone’s when our methods of teaching are made more individual and personal through good tutors. Then the tremendous appeal of social ethics to the spirit of our time can be presented with its full force.

 

Source: Harvard Alumni Bulletin, Vol. 23, No. 30 (May 5, 1921) pp. 688-689, pp. 700-702.

Image: Robert Franz Foerster, Assistant Professor of Social Ethics. In Harvard Class Album 1920.

Categories
Economists Harvard

Harvard. Taussig’s assessment of the French economist Charles Rist for a Harvard lectureship, 1919

 

 

After Edwin F. Gay resigned his position at Harvard, Abbott Payson Usher took over his courses in 1921-22. (e.g. Economics 2a: European Industry and Commerce in the Nineteenth Century). From the files of President Lowell of Harvard we find that the French economist Charles Rist was seriously considered for that position. Frank Taussig‘s brief letter, transcribed below, was apparently sufficient to get a green-light from the President’s Office. I don’t know (yet) what was the deal breaker or even whether an offer actually ever went out.

_______________

Letter of Economics Chairman E. E. Day to President Lowell

HARVARD UNIVERSITY
DEPARTMENT OF ECONOMICS

Cambridge, Massachusetts

March 4, 1920

Dear President Lowell:

I spoke to you some time ago of the Department’s wish that an invitation be extended to Professor Charles Rist to come as Lecturer in the Department for at least one half of the next academic year. I have not broached the subject again, because Mr. Gay has thought he might have other suggestions to make. It now appears that the expectations Mr. Gay had in mind will not materialize, and that he has no proposal to make which seems to him to promise better than that the Department had in mind. I consequently renew at this time the Department’s suggestion. In view of Mr. Gay’s resignation, the offering of the Department is obviously deficient. I understand that you will support the Department in its endeavor to discover a man who may be brought in permanently to fill in part the serious gap which Mr. Gay’s departure has created. The suggested invitation to Professor Rist is one of the measures in this direction which the Department thinks most promising.

Professor Taussig is the only member of the Department who has had an opportunity to become personally acquainted with Professor Rist. I enclose herewith a statement of Professor Taussig’s impressions of the man. The other members of the Department know Rist only through his publications. These appear to be of highest quality.

It is the proposal of the Department that an invitation be extended to Rist to lecture here during the first half of 1920-1921. Possibly he may be secured on an exchange arrangement. If not, the Department would like to see him appointed as Lecturer in Economics for not less than the first half of the year.

Sincerely yours,
[signed]
Edmund E. Day

Enc
President A. Lawrence Lowell

_______________

From a typed copy of Taussig’s statement

DEPARTMENT OF ECONOMICS

Cambridge, Massachusetts
November 28, 1919

            Professor Charles Rist is a member of the staff of the Sorbonne in the Department of Law. Economics is one of the subjects required of law students in France, hence there is a considerable economic staff for the law students. Rist is a man of 40-45 years, an extremely temperate, clear-headed, scholarly person. Of all the French professors with whom I came in contact in France he seemed to me the most promising. He has a most attractive personality, and is a clear as well as pleasing writer. His scholarly standing is assured. He is married, and has a family of several boys. For the sake of the boys, as well as for his own advantage, he remarked to me that he would very much like to come to the United States. If tolerable pecuniary arrangements can be made, he would doubtless come.

Rist’s command of English is not now sufficient to enable him to lecture in English. He would have to arrange to come over here a couple of months in advance and acquire a reasonable command of the spoken language. I should myself strongly advise him to do this, in case an invitation were extended.

Rist is the only man whom I saw in France who seemed to me a serious possibility for a permanent member of our staff. I think very highly of the man and his work, and have this possibility in mind in recommending him.

(Sgd) F.W. TAUSSIG

_______________

Copy of Lowell’s Response to E. E. Day

March 9, 1920

Dear Mr. Day:

It seems to me that the best thing would be to have Professor Rist sent here as the exchange professor from the University of Paris next year. We do not like to ask authoritatively to have a particular person sent, because we should not like it if they did the same to us. Therefore the best plan would be to have Professor Taussig write to him, suggesting that he should ask to be sent here next year as exchange professor, and he might add that he, M. Rist, feels confident that his selection would be acceptable at Harvard.

Very truly yours,
[name stamp] A. Lawrence Lowell

Professor E.E. Day
Department of Economics
Massachusetts Hall
Cambridge, Mass.

 

Source: Harvard University Archives, President Lowell’s Papers, 1919-1922, Box 155, Folder 293.

Image Source: Charles Rist at BnF Gallica website.

Categories
Uncategorized

MIT. Economics Ph.D. alumnus (1957) Jaroslav Vanek. Obituary, 2017

 

When I conducted a bit of scholarly due diligence to try to establish the date of a Harvard economics skit I posted a few days ago (the script refers to the fact that Professor Vanek was leaving the Harvard department much to the regret of the skit-writers), I discovered that MIT Ph.D. alumnus, Jaroslav Vanek, passed away last month. Having been raised a comparative economics economist, I was aware of some of his work and post the local obituary that provides some insight into the man and scholar. 

______________________

Vanek, Jaroslav
04/20/1930-11/15/2017
Published in Ithaca Journal on Nov. 18, 2017

Ithaca resident Jaroslav Vanek, 87 years of age, passed away peacefully at Hospicare of Ithaca on Wednesday, November 15, due to the effects of myelodysplasia. He was born in Prague, Czechoslovakia, on April 20, 1930, to Josef and Jaroslava Vanek. His father worked in the Czech government’s labor ministry. He survived the occupation of Czechoslovakia by the Axis forces from 1938 to 1945, and during this time his mother took up beekeeping as a way to make ends meet, which turned into an interest that he would continue in his later years. Other early activities included building a canoe to paddle on the Vltava River that passes through Prague, and time spent at the family’s summer house in the village of Voznice, in the forested hills about 25 miles south of the city.

Jaroslav graduated from the Prague Gymnasium (high school) in 1949, where he was the pole vaulter on the track and field team. Later in life he reconnected with his high school classmates and attended many reunions with them in the Czech Republic in recent years. In September 1949, the family fled for political reasons into Germany, where they at first landed in a refugee camp in Munich, before eventually settling in Geneva and Paris.

The hardships of the Second World War and the communist takeover of Eastern Europe instilled in Jaroslav a lifelong desire to make the world a better place, including at first social justice and eradication of poverty, and later protection of the environment and development of renewable energy. Desire to increase the welfare of ordinary people led to an interest in economics, and he earned a degree in the field from the University of Geneva in 1954. He stayed on in Geneva to work as a professional economist, and while there met Prof. Charles Kindleberger of MIT. In 1955 he left Geneva for Cambridge to pursue a doctorate in economics under Prof. Kindleberger’s supervision, which he earned in 1957. He then taught economics at Harvard University. In 1958 while teaching at Harvard he met Wilda Marraffino of Larchmont, NY, then a doctoral candidate in history at Harvard, and they married on December 26, 1959.

Jaroslav left Harvard for an economics position with the U.S. State Department in Washington, DC, in 1963, and then came to Ithaca in 1964 to take a position with the Department of Economics in the College of Arts and Sciences at Cornell. Upon arrival they rented a house from the late Prof. Herbert Gilman of Cornell’s Veterinary College at 414 Triphammer Road, and later purchased the house. Although he and Wilda traveled widely, they would always call the house, known among family members as either “Triphammer” or “414”, home for the rest of his days and years.

His early work focused on international economics, one notable book being “Maximal Economic Growth” published in 1968. In the late 1960s and early 1970s he turned his attention to labor managed cooperatives as an alternative to mainstream economic models, leading to titles such as “The Participatory Economy: An Evolutionary Hypothesis and a Strategy for Development” in 1971. He was visiting professor in Belgrade, Yugoslavia; Louvain, Belgium; Wassenaar, Netherlands; and The Hague, Netherlands. These appointments gave his growing family a chance to live and go to school in several foreign countries, which formed an influential part of their upbringing. He also advised the governments of Peru and Turkey during this time.

In 1979 while on his second sabbatical at the Institute for Social Studies in The Hague, he got to know a Bangladeshi graduate student named Shakur who was eager to start a cooperative using solar energy to bake clay building bricks. This encounter sparked Jaroslav’s lifelong interest in appropriate technology combined with cooperatives, as a way to address economic inequality in the world. On his return to Ithaca he started the “Ensol” solar energy cooperative and began involving his family and graduate students in a flurry of low-tech solar inventions ranging from very large inflated parabolic discs, simple parabolas to generate steam power and improving simple one-pot solar oven design.

Jaroslav would also develop prototypes to harness wind and wave power. In 1986, as the work continued and attracted interest from around the world, Jaroslav and Wilda created the S.T.E.V.E.N. Foundation not-for-profit (Sustainable Technology and Energy for Vital Economic Needs) to fund continuing research and outreach abroad. Longtime Ithacans may remember the shiny Mylar-lined parabolic solar collectors which were visible in front of their home at 414 Triphammer Rd in the eighties and nineties.

Jaroslav enjoyed physical activity whether building his inventions in the backyard, walking or biking to his office on Cornell campus or summertime swimming in the local parks. In 1988 at 58 years old he biked around Cayuga Lake. His mother had remained in Geneva, Switzerland, and took up beekeeping as she had done during the war, so over the years he would help her when visiting. His children distinctly remember dad loading the family Volkswagen bus within an inch of breaking the rear axle at the Migro supermarket in Geneva with bags of sugar for mixing sugar-water for the beehives. In retirement, he enjoyed pitching in with projects at his daughter Teresa’s farm, notably assisting in beginning the beekeeping operation there and passing the interest to the third and eventually fourth generation. He also helped three local children with building their homes: a straw-bale home for each Teresa and her brother Steven, and a cohousing home for their brother Francis in the Ecovillage at Ithaca Second Neighborhood.

In 1989, forty years after he escaped with his parents, Jaroslav was invited to present at an economic conference in what was still Czechoslovakia – just before the Velvet Revolution. He brought the family along to see his birthplace for the first time. Thankfully he was able to reconnect with many friends and family. In 1992 the rise and fall of communism in Czechoslovakia came full circle and the summer house in Voznice that had been confiscated was returned to Jaroslav and his brothers’ ownership. This sturdy house that bordered on forests and a large swimming pond became a home away from home during annual visits to Czech Republic. Jaroslav and Wilda (and frequently their children and grandchildren) enjoyed bicycling, walking in the nearby forests – sometimes collecting delicious mushrooms and berries — and the cultural riches of Prague. They were fortunate to revive friendships in Czech Republic that continue until this day.

In closing, Jaroslav truly lived the maxim to “do all the good you can, for as long as you can.” He is survived by his wife of 57 years Wilda Vanek, and children and grandchildren: Josef Vanek, MD, surgeon in Uniontown, PA (wife Sue, children Carolyn and Tess); Francis (wife Catherine Johnson, children Ray and Mira), faculty at Cornell University; Rosie (husband Jon Liden), the Global Fund, Geneva, Switzerland; Steven (wife Leia Raphaeledis, children Anais and Jan), faculty at Colorado State University, Fort Collins; and Teresa (husband Brent Welch, children Milan and Luka), co-owner of Bright Raven Farm and Apiary.

A mass of Christian burial will be held at Immaculate Conception Church, 113 N. Geneva St., Ithaca, on Saturday, November 18, 2017 at 11:30 AM. Family will receive friends at the church at 10:45 AM. Burial will take place at Pleasant Grove Cemetery, in Cayuga Heights, following a brief reception after mass.

 

http://www.legacy.com/obituaries/theithacajournal/obituary.aspx?page=lifestory&pid=187271566

Categories
Exam Questions Harvard Suggested Reading Syllabus

Harvard. Undergraduate International Trade. Enrollment, Readings, Exam. Harris, 1949

 

Seymour Harris was a Harvard man from his undergraduate years through his retirement from his alma mater. He served many terms as a government economic adviser and after Harvard moved on to be the chair of economics at the University of California, San Diego. This post provides a course description, enrollment data, reading list and examination questions for his winter semester course 1949-50 “International Trade”. An earlier post provides the outline the 1933 version of the course “International Trade and Tariff Policies”.

Seymour Edwin Harris was born September 8, 1897 in New York City. He received an A.B. in 1920 and a Ph.D. in 1926 from Harvard University. From 1922 to 1964, Dr. Harris taught economics at Harvard University, where he received a full professorship in 1954, and served as the chairman of the department of economics from 1955 to 1959. During World War II, Dr. Harris was involved in several wartime planning projects. From 1954 to 1956, Dr. Harris became chief economic advisor to Adlai Stevenson. He then served Senator John F. Kennedy in the same capacity and was chosen as a member of President Kennedy’s task force on the economy. In 1961, Dr. Harris was named as chief economic consultant to Douglas Dillon, Secretary of the Treasury. During the Kennedy administration. Dr. Harris, a proponent of Keynesian economics, was a member of Walter W. Heller’s New Frontiersmen, which persuaded President Kennedy that the stimulation of the economy was more important than a balanced budget and tax cuts and government spending could counter threats of a recession. In 1963, Dr. Harris became the chairman of the department of economics at the University of California at La Jolla. At the same time, he served as a chief economic advisor to the Johnson administration. [Source: John F. Kennedy Presidential Library, Finding Aid to the Papers of Seymour E. Harris]

Harris had quite a reputation for grinding out volumes of edited papers (written by others):  From a 1962 or 1963 student skit:  Professor Gerschenkron’s alleged advice for Matthew (the Evangelist): “I wanted Matthew to rewrite his paper for the Quarterly Journal and call it ‘Christ as a proto-Keynsian’ [sic] But no, he was a very strong-willed boy and he brought it out in a syposium [sic] edited by Seymour Harris, called the Bible, essays in honor of God.”

_________________________

Course Description

Economics 143a (formerly Economics 43a). International Trade

Half-course (fall term). Mon., Wed., and (at the pleasure of the instructor) Fri., at 12. Professor Harris.

This course deals with the theory and practice of foreign trade and capital movements, including the importance of foreign trade, the manner of increasing its amount, its relation to the domestic economy, the problem of exchange rates, exchange control, international organizations in their relation to trade and capital movements. Political, economic, and administrative aspects are considered also.

Source: Harvard University Archives. Box 6, Courses of Instruction (HUC 8500.16), Final Announcement of the Courses of Instruction Offered by the Faculty of Arts and Sciences for the Academic Year 1948-49, p. 75

_________________________

Course Enrollment

[Economics] 143a (formerly Economics 43a). International Trade. (F) Professor Harris.

Total 120: 9 Graduates, 46 Seniors, 36 Juniors, 11 Sophomores, 1 Freshman, 12 Radcliffe, 5 Others.

Source: Harvard University. Report of the President of Harvard College and Reports of Departments, 1949-1950, p. 43.

_________________________

Course Readings

Economics 143a
International Trade

  1. National Income and the Balance of Payments—4 weeks
    Relation of domestic policies and the balance of payments; the balance of payments of the United States and the United Kingdom; capital movements and reparations problems; dollar shortage and the E.R.P.
    Assignment

    1. *Harris; The European Recovery Program, pp. 1-120, 185-206, 272-276
    2. *Harrod; Are These Hardships Necessary? pp. 1-103
    3. United Nations; A Survey of the Economic Situation and Economic Prospects for Europe, pts. 2-5
  2. Regional Problems in the Balance of Payments—1 week
    No assignment.
  3. Industrialization and International Competition—1 week
    No assignment
  4. Monetary Aspects of International Trade—3 weeks
    Assignment

    1. Harris: The New Economics, pp. 246-293, 323-400
    2. *Ellsworth: International Economics, Part 1, Chs. 7, 9-11
  5. The Case for Free Trade and Obstacles to Trade—3 weeks
    Division of labor; comparative costs; tariffs and other obstacles; international commodity agreements and distribution of raw materials.
    Assignment
    Ellsworth, Part II, Chs. 1-5, 7-9
  6. The Problem of Allocation of Resources and Comparative Costs
    Assignment
    Ellsworth, Part I, Chs. 3-5

Reading Period Assignment—One of the Following:

  1. Staley: World Economic Development
  2. Buchanan and Lutz: Rebuilding the World Economy
  3. Marshall: Money, Credit, and Commerce, Pt. III
  4. Harris: Foreign Economic Policy for the United States

*To be bought.

Source:   :   Harvard University Archives. Syllabi, course outlines and reading lists in economics, 1895-2003. (HUC 8522.2.1), Box 4, Folder “Economics, 1949-50 (2 of 3)

_________________________

Course Final Examination

HARVARD UNIVERSITY
ECONOMICS 143a
International Trade

Spend forty-five minutes on each question. It is strongly suggested that you spend ten minutes assembling and organizing your thoughts before beginning each essay.

  1. Answer (a) or (b):
    1. It is a prerequisite for the establishment of interregional trade that relative prices would be different in one region from those in another if the regions were prevented from trading with one another. Why does this condition commonly occur, how does it lead to interregional trade, and in what way is that trade beneficial to the participating regions?
    2. Much of the theory of international trade attempts to explain how equilibrium in the balance of payments will be maintained by automatic tendencies. Present as fully as you can in the allotted time a classification and explanation of the forces working to frustrate these automatic tendencies and produce persistent dis
  2. Answer (a) or (b):
    1. Exchange depreciation has been resorted to in depression for reasons very different from those advanced for its use in periods of full employment. Compare and contrast the advantages and disadvantages of depreciation in these two situations, making use of specific illustrations.
    2. The incorporation of national income analysis into international trade theory has led economists to distinguish helpfully corrective international economic policies from so-called beggar-thy-neighbor policies. How would you make such a distinction, and how effectively can it be applied?
  3. Answer (a) or (b):
    1. Comment on the relevance of the domestic policies of ERP countries for the attainment of international equilibrium by 1952.
    2. Discuss the reciprocal trade agreements program of the United States with reference to (1) the general arguments for and against tariffs, and (2) the limitations of tariff policy as a means of achieving international economic equilibrium.
  4. Answer one of the following with reference to your reading period assignment:
    1. What basic changes in the world economic structure have resulted from two World Wars, and how do they obstruct the reestablishment of stable multilateral trade?
    2. On what conditions does Staley base his case that, for existing industrial areas, it is possible to make the advantages of the economic development of new areas far outweigh the disadvantages? Discuss these conditions critically.
    3. Diagnoses of the widespread “dollar problem” differ according to (1) the definition of equilibrium adopted, and (2) the particular country under discussion. Discuss both these sources of variation in analysis. (Treat either one at greater length than the other if you wish.)
    4. Marshall is noted for his development of the theory of international supply and demand (commonly called “reciprocal demand”) as the determining influence on the barter terms of trade. Develop this part of his theory, and comment on some aspect or aspects of its relevance to current problems discussed in this course.

Final. January, 1950.

 

Source: Harvard University, Faculty of Arts and Sciences. Papers Printed for Final Examinations; History, History of Religions, Government, Economics,…,Military Science, Naval Science. February, 1950.

Image Source: Seymour Harris in Harvard Class Album 1947.

 

Categories
Funny Business Harvard M.I.T.

Harvard or MIT. Economics graduate student skit, ca. 1963.

 

Because of the reference to Jaroslav Vanek’s leaving Harvard, we are able to date the following script to 1962-63 since Vanek left Harvard to work at the State Department in 1963. Almost everything about this script would lead me to conclude that it was used in a Harvard graduate student skit that somehow wound up in the folder for the Graduate Student Association at the Department of Economics of M.I.T. The folder is otherwise filled with clearly M.I.T. skit material from the 1960s. One of the students is identified as “David” another “Bob” and the third looks like “Les”.  

Lester Thurow did get his Ph.D. from Harvard in 1964 and came to M.I.T. in 1968 so it is not inconceivable that the following transcription is indeed based upon his personal typed script copy with original pencil stage directions that made its way into the folder. 

One thing that I find rather surprising about the text is just how many Harvard professors’ names have been misspelled.

__________________________

D—This is a review with a message—a message no economist can afford to ignore. The year is 2000 A.D. 16 years have now passed since 1984, that Armageddon of the economics profession when Professor Wassily Leontief finally established that the world really was homogeneous of degree one. The then President of the United States, Mr. Norman Mailer, immediately issued the great Marginal Product Proclamation. Everyone was to receive their marginal product.

B— But there was nothing left over for the economists. Economists became the hand-loom weavers of the 20th. century.

L—Arthur Schlesinger Jr. vividly described their position in a 17-volume work entitled “The Coming of the Raw Deal.” Economists everywhere, after the first shock, set out upon new careers. Tonight we shall discover what happened to some of those whom we know and love.

D—Several of them went into the movie industry and we will now let you hear the soundtrack of the preview of one of their movies.

(Epic Music—Bruckner?)

[Insert: Stand]

L—Ladies and Gentlemen, 21st Century Fox are proud to present Arthur Smithies and Joan Robinson in….The Big Push, the story of the unbalanced growth of an economist….

B—Production by Karl [sic] Kaysen

D—Copyright by Edward Hastings Chamberlain [sic]

L—All labor disputes on location and with Elizabeth Taylor arbitrated by John Dunlop.

B—Continuity by Simon Kuznets

L—Editing by Seymour Harris, of course.

D—Costumes by Robert Dorfman.

B—This is the story of Ragnar Maynard von Eckstein (his parents had always wanted him to be an economist). After many struggles at last he got to Harvard Graduate School.

L—It is a tale of |horror. See him now at a seminar on the economics of Medical Care…..

D—This after-noon I am going to discuss the economics of Blood-banking. One of the crucial problems in this field is what proportion to maintain of liquid assets. In this category we have blood [Insert:   L. What about near blood] near-blood. We also have non-liquid assets—bonds in the form of pounds of flesh. Another problem is the current shortage of tellers, for we can only employ vampires with a strong liquidity preference. If we cannot get more it will clot up the flow of funds and reduce the velocity of circulation.

L—It is a tale of |ambition…..

B—Coming from a family whose marginal product was zero, Ragnar Maynard realized that to get on quickly he must publish something. But what? He had not written anything. But our resourceful hero saw a way out: he would publish his first book before it was written. It was called First Draft, a revised tentative, preliminary, provisional text. It was based on Photostat copies of his blackboard notes.

L—It is a tale of |love….

D—Ragnar Manyrd fell passionately in love with a beautiful capital theorist, played in the movie by ravishing Joan Robinson. His demand for her love was infinitely elastic; her supply could not meet him—at least not at his price. The price was to join him in his exhausting search over peaks and through troughs for the elusive U-shaped cost curve.

L—It is a tale of |excitement

B—See Ragnar Maynard trying to free himself from the dreaded liquidity trap.

Insert: D—It’s true, it really is thicker than water

L—All this and more you can see in this movie—The Big Push is a take-off point in the development of the motion-picture.

B—See the exciting attempt on Professor Leontief’s life (with a 202 rifle) to try to prevent him revealing his startling discovery of a constant returns world.

D—See the world’s largest input-output table which proved it—drawn by the Economic Research project in the sand of the New Mexican desert.

L—You cannot afford to miss this motion picture. Filmed in wonderful new—Solocolor. An introducing revolutionary—Rostowscope.

(concluding epic music)

[Insert: Sit]

D—But the movies could not accommodate everybody…

[Insert: Bob in middle]

[Insert: one illegible word]

L—Professor Leontief, having escaped with his life, and using his input-output table from Scientific American as a testimonial, got into the business of designing bathroom tiles.

B—Professor Duesenbery [sic] was well qualified to go into the demonstration business. He drove Cadillacs around low-income districts to stimulate demand. And changed his name to Jones so that it would be him that everyone was keeping up with.

D—In England many economists went to work for the government where they produced a remarkable effect. Before 1984 political speeches had sounded something like this.

B—Good evening; I’m the Prime Minister. My name is….. [insert: ad lib] etc.

D—But now all this has changed…

B—Good evening…[insert: ad lib] etc.

L—Professor Tom Schelling took up a career in Madison avenue. It was he who was responsible for some of the following products…

D—Ladies, now you can wear the most powerful and alluring perfume in the world—First Strike—the only perfume with complete credibility. It also contains the only deodorant with overkill.

B—Now at last there is a product to take away the smell of deodorant—it is called Counterforce. Only Counterforce gives you 24-hour protection against odorlessness. [Insert: 5120 or S120]

[Insert: STAND]

L—For years girls have been searching for a perfume which will attract the men and yet prevent them from taking liberties—now they have it in the form of Deterrence—the perfume which is effective [Insert: only] if you don’t use it.

D—He also introduced a city wide deodorant campaign under the title of Civil defence.

L—And the only really safe method of birth control—Early Warning.

B—Meanwhile Professor Dunlop had become a truck driver and a shop steward for Jimmy Hoffa.

D—And Professor Kuznets took to selling abacuses.

[Insert: Some economists, not from Harvard opened a cafeteria.]

[Insert: Bob-Les—come forward]

L—Professor Galbraith first thought of becoming a rice farmer. But he soon saw that since there was no more need for economists he could now come into his own. After a coup d’etat he took over the Littauer building and changed it into the department of Affluent Studies. The idea was the ultra-popularization of economics; the main qualification for admission was to be a good phrase-monger. The new department published books like…

B—The Economics of Sex, with an appendix on the second derivatives of Jayne Mansfield. A geometric interpretation with diagrams.

D—The department became identified with a new theory of economic decline, published as a non-Rostovian manifesto. All countries, it said, tend to decline, and their speed of decline is determined by their relative degree of economic advancement. Its five stages of decline started with the age of mass consumption, through the age of preconditions for decline, coming then to the crucial landing stage.

B—Other books appeared like ‘The Naked Truth about Public Squalor, and so on.

[Insert: Pause—back to audience]

L—Only one of the redundant economists took the highest calling of all. Let us now eavesdrop on a sermon by [Insert: his eminence] Archbishop Gerschenkron…

[Insert: seated]

B—You know, when I was an economist one of my graduate students wrote a very good paper for my course. Matthew, [Insert: I said] why don’t you publish this paper, no, really why don’t you publish. But you know youll have to change the title. What journal is going to publish a paper called ‘the First Gospel’? But you know it really was a very good paper. There was a lot of interesting material about the farm problem in Egypt and about the almost miraculous elasticity of supply of loaves and small fishes in Gallillee [sic]. Then there was a very good section about Christ throwing the money-changers from the temple. Well, you see, the rate of interest was very high then. Don’t you think that the real reason why Christ did this was to reduce the rate of interest and to stimulate investment. You see, I wanted Matthew to rewrite his paper for the Quarterly Journal and call it ‘Christ as a proto-Keynsian’ [sic] But no, he was a very strong-willed boy and he brought it out in a syposium [sic] edited by Seymour Harris, called the Bible, essays in honor of God. But, you know, it was still required reading for my course.

D—Professor Harberler [sic] took to song writing, and here is a sample…

[Insert: stand behind table]

(tune: God bless America)

[Insert: All:] God bless free enterprise,
[Insert: MOC or HOC or NOC] System divine,
Stand beside her and guide her,
Just as long as the profits are mine.
[Insert: Salute]
Corporations may they prosper
Big business, may it grow!
[Insert: MOC or HOC or NOC] God bless Free Enterprise,
The Status quo!

L—Well, David, I guess that’s it. Do you think they’ll throw us out?

D—I dont know. But I dont suppose we’ll ever be allowed to pass generals. There are still some jobs you can get without a Ph.D.

B—No chance at all is there? I mean about generals….

D—Well they were all in it weren’t they—all the generals board.

L—What about Professor Vanek? He emerged unscathed.

D—That’s true but he’s leaving.

B—That’s fair, of course.

L—Yes, he hasn’t done much since he’s been here really.

D—Half a dozen good articles…

B—4 books, or is it 5?

L—He’s become an acknowledged expert on at least two major fields of economics…

D—A clear and stimulating teacher…
And a nice guy…

L—Not much really. [Insert: Clearly not a Harvard type]

B—Not surprised they’re letting him go

D—Well, that’s it then.

B—One more thing actually…The perpetrators of this entertainment would like it to be known that any resemblance of characters in this review to any person or persons living or half-dead is purely intentional.

L—So be it.

All—In the name of the Holy Trinity:

D—Dorfman,

L—Samuelson,

B—and Solow.

All—Amen

 

Source:   MIT Archives. Department of Economics Records, Box 2, Folder “GEA 1961-67”.